Magnetic force between 2 moving charges If 2 positive charges are moving in the same direction wit

affizupdaftf3opg

affizupdaftf3opg

Answered question

2022-05-19

Magnetic force between 2 moving charges
If 2 positive charges are moving in the same direction with a velocity v relative to the ground frame, will they experience a magnetic force? I’ve read several times that the velocity in the Lorentz force law is relative to the ground frame, not to the object creating the magnetic field, so I feel like there should be a magnetic force.
Moreover, assuming there is a force, I tried computing the velocity at which the electric and magnetic force cancel each other, and got the speed of light. Is this just a coincidence, or is there any proper reasoning behind this?

Answer & Explanation

Astok3mpd

Astok3mpd

Beginner2022-05-20Added 18 answers

According to "Darwinian Lagrangian - Interacting particles" the force between two charged particles (written in SI units) is
F = 1 4 π ϵ 0 q 1 q 2 r 2 r ^ + μ 0 4 π q 1 q 2 r 2 1 2 { v 1 ( r ^ v 2 ) + v 2 ( r ^ v 1 ) r ^ [ v 1 v 2 + 3 ( v 1 r ^ ) ( r ^ v 2 ) ] }
where q 1 and q 2 are the charges, v 1 and v 2 are the velocities of the two prticles, and r ^ is the unit vector between them.
The first term is the well-known electric Coulomb force between the two particles.
The second (very long) term is a magnetic force between the two particles. Roughly speaking, this magnetic force is attractive when q 1 v 1 and q 2 v 2 are parallel. And it is repulsive when they are antiparallel.
But remember that this Darwinian formula is still an approximation valid only for slow velocities ( v 1 , v 2 c). So don't expect correct results when v 1 c or v 2 c
Having said that, you see when both v 1 and v 2 approach c, then the magnetic force becomes similar in size to the Coulomb force. (Remember μ 0 = 1 ϵ 0 c 2 to verify this.)
Micah Haynes

Micah Haynes

Beginner2022-05-21Added 2 answers

The real question that needs to be asked is this: "according to whom?" The special theory of relativity states that all inertial observers' points of view of the world are equivalent.
According to an observer in the laboratory frame, charges moving at a constant velocity possess both an electric and a magnetic field. The magnetic field of such a moving charge can be calculated without too much difficulty, and it can be shown that if we know the moving charge's electric field E , then
B = v c 2 × E .
Thus, an observer in the lab will say that each of the moving charges is affected by force due to both the electric and magnetic fields of the other charge.
However, if we looked at this from the point of view of one of the charges (or perhaps an observer moving with the same speed as the charges), both these charges would appear to be at rest with respect to each other, and therefore there would be no magnetic field there. The charges would interact with a purely electric field, E , which is different from E
As I've shown in the linked answer, you can compute the electric and magnetic fields experienced by the second charge due to the first as show that:
F E = γ F E  and F B = γ v 2 c 2 F E F lab = γ ( 1 v 2 c 2 ) F E = F E γ ,
where γ = 1 / 1 v 2 / c 2 , and F E is the Coulomb force in the particle's rest frame.
Using the above relation, you should also be able to prove why imposing the magnetic and electric forces being the same on each of the charges leads to v = c. (See the end of the answer I linked.) I doubt it's a coincidence, but I think it needs to be interpreted as saying that the force due to the magnetic field of a moving charge can never be equal to the force due to its electric field or greater than it (since v can never be equal to c for a massive object).
A simple reason for this I can think of is the following: as I've shown above, the force due to the magnetic field is opposite in direction to the force due to the electric field. (Like charges repel, but like "currents" attract.) Now imagine that there could be some constant speed v such that their magnitudes could be equal. This would mean that both these forces would cancel each other out; i.e. there would be some "privileged" speed v where the particle would not experience a net force. But this violates the very idea of an inertial frame! Therefore it must not be possible for any sensible v. And indeed, using the formula for F lab above, you can see that the measured force seems to reduce, but it never goes to zero for, since v < c. So I suppose -- for what it's worth -- you could say that this is "due" to special relativity.

Do you have a similar question?

Recalculate according to your conditions!

New Questions in Electromagnetism

Ask your question.
Get an expert answer.

Let our experts help you. Answer in as fast as 15 minutes.

Didn't find what you were looking for?